LSAT and Law School Admissions Forum

Get expert LSAT preparation and law school admissions advice from PowerScore Test Preparation.

User avatar
 Dave Killoran
PowerScore Staff
  • PowerScore Staff
  • Posts: 5850
  • Joined: Mar 25, 2011
|
#44131
Complete Question Explanation
(The complete setup for this game can be found here: lsat/viewtopic.php?t=8628)

The correct answer choice is (D)

If Q and R are assigned to the cages, then S must be exhibited. If S is exhibited, then from the third rule J and W must be assigned to the cages. Accordingly, answer choice (D) is correct.

Get the most out of your LSAT Prep Plus subscription.

Analyze and track your performance with our Testing and Analytics Package.